Math, asked by BhumiBakshi, 4 months ago

A shopkeeper sold two watches for ₹920 each. On one watch he gained 15% and on the other he lost 60%. Fond his profit or loss percent.

need answer in process
please help me it's emergency​

Answers

Answered by prince5132
43

GIVEN :-

  • A shopkeeper sold two watches for Rs. 920 each.

TO FIND :-

  • His profit or loss %.

SOLUTION :-

 \underline{ \bigstar  \: \textsf{Situation \: 1.}} \\  \\

 \bullet \displaystyle \sf \: S.P = Rs. 920 \\  \\

\bullet \displaystyle \sf \: profit \: \% = 15 \\  \\

 : \implies\displaystyle \sf \: C.P =  \frac{S.P \times 100}{100 + profit \: \%}  \\  \\  \\

: \implies\displaystyle \sf \: C.P \:  =  \frac{920 \times 100}{100 + 15}  \\  \\  \\

: \implies\displaystyle \sf \: C.P \:  =  \frac{92000}{115}  \\  \\  \\

: \implies \underline{ \boxed{\displaystyle \sf \: C.P \:  = Rs. \: 800}} \\  \\

\underline{ \bigstar  \: \textsf{Situation \: 2.}} \\  \\

\bullet \displaystyle \sf \: S.P = Rs. 920 \\  \\

\bullet \displaystyle \sf \: loss \: \% = 60 \\  \\

: \implies\displaystyle \sf \: C.P =  \frac{S.P \times 100}{100  -  loss \: \%}  \\  \\  \\

: \implies\displaystyle \sf \: C.P \:  =  \frac{920 \times 100}{100 - 60}  \\  \\  \\

: \implies\displaystyle \sf \: C.P \:  =  \frac{92000}{40}  \\  \\  \\

: \implies\displaystyle \sf \: C.P =  \frac{9200}{4}  \\  \\  \\

: \implies \underline{ \boxed{\displaystyle \sf \: C.P \:  = 2300}} \\  \\

_______________…

 \\ \dashrightarrow\displaystyle \sf \:total \:  C.P = 2300 + 800 \\  \\  \\

\dashrightarrow \underline{ \boxed{\displaystyle \sf \:total \:  C.P  = 3100}} \\  \\

Now,

 \\ \dashrightarrow\displaystyle \sf \:total \:  S.P  = 920 + 920 \\  \\  \\

\dashrightarrow \underline{ \boxed{\displaystyle \sf \:total \:  S.P   = 1840}} \\  \\

______________…

 \\   : \implies \displaystyle \sf \: S.P  < C.P \\  \\  \\

: \implies \displaystyle \sf \:loss \:  = C.P - S.P \\  \\  \\

 : \implies \displaystyle \sf \:loss = 3100 - 1840 \\  \\  \\

 : \implies\underline{ \boxed{\displaystyle \sf \:loss = 1260}} \\  \\

Now,

 \\  : \implies \displaystyle \sf \:loss \: \% =  \frac{loss}{C.P}  \times 100 \\  \\  \\

 : \implies \displaystyle \sf \:loss \: \% =  \frac{1260}{3100}  \times 100 \\  \\  \\

 : \implies \displaystyle \sf \:loss \: \% =  \frac{1260}{31}  \\  \\  \\

 : \implies \underline{ \boxed{ \displaystyle \sf \:loss \: \% = 40.64}}

Answered by diajain01
26

{\boxed{\underline{\tt{ \orange{Required  \:  \: Answer:-}}}}}

★GIVEN:-

  • A shopkeeper sold two eatches at Rs. 920.

  • At one watch profit % is 15%

  • At another watch loss % is 60%

★TO FIND :-

  • his profit or loss percent.

★SOLUTION:-

1st CASE:-

: \implies \:  \tt{S.P.\: = \:Rs. \:920}

: \implies\tt{Profit\% = 15\%}

: \implies \:  \tt{C.P.  =  \frac{s.p. \times 100}{100 + p\%}}

: \implies \:  \tt{C.P.\: =\: \frac{920×100}{100+15}}

: \implies \:  \tt{C.P. \: =\: \frac{92000}{115}}

: \implies \:  \tt{C.P.\:=\: Rs.800}

2nd CASE:-

: \implies \:  \tt{S.P.\:=\:Rs.\:920}

: \implies \:  \tt{Loss\% = 60\%}

: \implies \:  \tt{C.P.  =  \frac{s.p. \times 100}{100  - loss\%}}

: \implies \:  \tt{C.P. \: = \: \frac{920×100}{100-60}}

: \implies \:  \tt{C.P. \: = \: \frac{92000}{40}}

: \implies \:  \tt{C.P. \: = \: \frac{1200}{4}}

: \implies \:  \tt{C.P.\:=\: Rs.300}

_______________________________________________★

TOTAL C.P. = 2300 + 800

: \implies \:  \:  \:  \:  \:  \:  \:  \:  \:  \:  \:  \:  \:  \:  \:  \:  \:  \:  \:  \:  \:  \:  \:  \:  \:  \:  \: {\boxed{\underline{\tt{TOTAL \:  \:  C.P. = Rs.\:\:3100}}}}

TOTAL S.P. = 920 + 920

: \implies \:  \:  \:  \:  \:  \:  \:  \:  \:  \:  \:  \:  \:  \:  \:  \:  \:  \:  \:  \:  \:  \:  \:  \:  \:  \: {\boxed{\underline{\tt{TOTAL \:  \:  S.P. = Rs.\:\:1840}}}}

SO,

S.P. < C.P.

Loss = 3100 - 1840

: \implies \: {\boxed{\underline{\tt{loss = Rs.\:\:1260}}}}

: \implies \:  \tt{Loss\% =  \frac{loss}{C.P}  \times 100}

: \implies \:  \tt{Loss\% =  \frac{1260}{3100}  \times 100}

: \implies \:  \tt{Loss\% =  \frac{12600}{31}}

: \implies \:{\boxed{\underline{\tt{loss\% = 40.64\%}}}}

HOPE IT HELPS

Similar questions